LSAT and Law School Admissions Forum

Get expert LSAT preparation and law school admissions advice from PowerScore Test Preparation.

 Administrator
PowerScore Staff
  • PowerScore Staff
  • Posts: 8915
  • Joined: Feb 02, 2011
|
#41662
Complete Question Explanation
(The complete setup for this game can be found here: lsat/viewtopic.php?t=8572)

The correct answer choice is (B)

This question asks us to identify the number of possible solutions if W is shown first. A quick look at our templates reveals that only Templates 2A and 3A allow W to be first. Once W is firmly in place in each template, the placement of the remaining two variables– R and S—will also be fully determined, in compliance with the third rule:
PT65_D11 LG Explanations_game_#4_#18_diagram 1.png

Since there are two ways in which we can order the programs, answer choice (B) is correct.
You do not have the required permissions to view the files attached to this post.

Get the most out of your LSAT Prep Plus subscription.

Analyze and track your performance with our Testing and Analytics Package.